The probability of picking a 20 dollar bill is 1/7 or 14.3%
How do we calculate for the probability of picking up a 20 dollar bill?The probability of a thing is the likelihood or number of chances that such a thing will occur. For the scenario given,
There are a total of 7 bills in your pocket
1, 1, 1, 1,
5, 5,
20.
To find the probability of picking out the twenty dollar bill, divide the number of twenty dollar bills by the the total of the number of bills you are with.
Probability = 1/ 7 which can be converted to % = 14.3%.
Find more exercises on probability;
https://brainly.com/question/11234923
#SPJ4
Please help!!! you are painting the roof of a shed that is 35 ft from the ground. you are going to place the base of a
ladder 12 ft from the shed. how long does the ladder need to be to reach the roof of the shed? use pencil and
paper. explain how shortening the distance between the ladder and the shed affects the height of the ladder. the ladder needs to be ____ ft long to reach the roof of the shed.
To find the length of the ladder needed to reach the roof of the shed that is 35 ft from the ground with the base of the ladder 12 ft from the shed, you can use the Pythagorean theorem. The theorem states that in a right-angled triangle, the square of the length of the hypotenuse (the ladder, in this case) is equal to the sum of the squares of the other two sides (the height and the distance from the shed).
Step 1: Identify the sides of the triangle.
- Height (a): 35 ft (vertical side)
- Distance from the shed (b): 12 ft (horizontal side)
- Ladder length (c): Hypotenuse
Step 2: Apply the Pythagorean theorem.
- a² + b² = c²
- 35² + 12² = c²
Step 3: Calculate the squares and sum them.
- (35 * 35) + (12 * 12) = c²
- 1225 + 144 = c²
- 1369 = c²
Step 4: Find the length of the ladder (c).
- c = √1369
- c = 37
The ladder needs to be 37 ft long to reach the roof of the shed.
Shortening the distance between the ladder and the shed will affect the height of the ladder by making it steeper. This will cause the ladder to be higher above the ground, but it may also make it less stable and more difficult to climb.
To know more about Pythagorean theorem refer here
https://brainly.in/question/47213677#
#SPJ11
What is the measure of ∠ABC?
Find the critical points for the function f(x, y) = x³ + y³ – 9x² – 3y - 6 = and classify each as a local maximum, local minimum, saddle point, or none of these. critical points: (give your points as a comma separated list of (x,y) coordinates.) classifications: (give your answers in a comma separated list, specifying maximum, minimum, saddle point, or none for each, in the same order as you entered your critical points)
The critical points and their classifications are:
(0, 1) - saddle point
(0, -1) - saddle point
(6, 1) - local minimum
(6, -1) - local minimum
To find the critical points of the function f(x, y) = x³ + y³ – 9x² – 3y - 6, we need to find the points where the partial derivatives of f with respect to x and y are zero.
∂f/∂x = 3x² - 18x = 3x(x - 6)
∂f/∂y = 3y² - 3 = 3(y² - 1)
Setting these partial derivatives equal to zero and solving for x and y, we get:
x = 0 or x = 6
y = ±1
So the critical points are (0, 1), (0, -1), (6, 1), and (6, -1).
To classify each critical point, we need to compute the second partial derivatives of f:
∂²f/∂x² = 6x - 18
∂²f/∂y² = 6y
∂²f/∂x∂y = 0
At (0, 1):
∂²f/∂x² = -18 < 0 (concave down)
∂²f/∂y² = 6 > 0 (concave up)
So (0, 1) is a saddle point.
At (0, -1):
∂²f/∂x² = -18 < 0 (concave down)
∂²f/∂y² = 6 > 0 (concave up)
So (0, -1) is a saddle point.
At (6, 1):
∂²f/∂x² = 18 > 0 (concave up)
∂²f/∂y² = 6 > 0 (concave up)
So (6, 1) is a local minimum.
At (6, -1):
∂²f/∂x² = 18 > 0 (concave up)
∂²f/∂y² = 6 > 0 (concave up)
So (6, -1) is a local minimum.
For more such questions on Critical points.
https://brainly.com/question/30889798#
#SPJ11
10 foot ladder is leaning against a vertical wall when Jack begins
pulling the foot of the ladder away from the wall at a rate of 0.5
fr/s. how fast is the top of the ladder sliding down the wall?
We can use the Pythagorean theorem to relate the distances between the ladder, wall, and ground. Let's call the distance from the foot of the ladder to the wall "x", and the distance from the top of the ladder to the ground "y". Then, we know that:
x^2 + y^2 = 10^2
We can differentiate this equation with respect to time to get:
2x(dx/dt) + 2y(dy/dt) = 0
We're interested in finding dy/dt, the rate at which the top of the ladder is sliding down the wall. We know that dx/dt = 0.5 ft/s, so we can plug in these values and solve for dy/dt:
2x(dx/dt) + 2y(dy/dt) = 0
2(8)(0.5) + 2y(dy/dt) = 0 (since x = 8 based on the Pythagorean theorem)
dy/dt = -4 ft/s
So the top of the ladder is sliding down the wall at a rate of 4 ft/s.
When the 10-foot ladder is leaning against a vertical wall, it forms a right-angled triangle with the wall and the ground. As Jack pulls the foot of the ladder away from the wall at a rate of 0.5 ft/s, the top of the ladder slides down the wall. To find the rate at which the top of the ladder slides down, we can use the Pythagorean theorem:
a^2 + b^2 = c^2
where a is the distance from the foot of the ladder to the wall, b is the height of the ladder's top from the ground, and c is the length of the ladder (10 feet).
Differentiating both sides with respect to time (t), we get:
2a(da/dt) + 2b(db/dt) = 0
We know that da/dt = 0.5 ft/s. We need to find db/dt, which is the rate at which the top of the ladder slides down the wall. To do this, we need to find the values of a and b at a given moment. Since the problem doesn't provide this information, it's not possible to determine the exact value of db/dt. However, if you have the values of a and b, you can plug them into the equation and solve for db/dt.
learn more about Pythagorean theorem here: brainly.com/question/343682
#SPJ11
Tom Jones, a mechanic at Golden Muffler Shop, is able to install new mufflers at an average rate of 3 per hour (exponential distribution). Customers seeking this service, arrive at the rate of 2 per hour (Poisson distribution). They are served first-in, first-out basis and come from a large (infinite population). Tom only has one service bay.
a. Find the probability that there are no cars in the system.
b. Find the average number of cars in the system.
c. Find the average time spent in the system.
d. Find the probability that there are exactly two cars in the system
a. To find the probability that there are no cars in the system, we need to use the formula for the steady-state probability distribution of the M/M/1 queue:
P(0) = (1 - λ/μ)
where λ is the arrival rate (2 per hour) and μ is the service rate (3 per hour).
P(0) = (1 - 2/3) = 1/3 or 0.3333
Therefore, the probability that there are no cars in the system is 0.3333.
b. To find the average number of cars in the system, we can use Little's Law:
L = λW
where L is the average number of cars in the system, λ is the arrival rate (2 per hour), and W is the average time spent in the system.
We can solve for W by using the formula:
W = 1/(μ - λ)
W = 1/(3 - 2) = 1 hour
Therefore, the average number of cars in the system is:
L = λW = 2 x 1 = 2 cars
c. To find the average time spent in the system, we already calculated W in part b:
W = 1 hour
d. To find the probability that there are exactly two cars in the system, we need to use the formula for the steady-state probability distribution:
P(n) = P(0) * (λ/μ)^n / n!
where n is the number of cars in the system.
P(2) = P(0) * (λ/μ)^2 / 2!
P(2) = 0.3333 * (2/3)^2 / 2
P(2) = 0.1111 or 11.11%
Therefore, the probability that there are exactly two cars in the system is 11.11%.
To know more about Poisson's Distribution:
https://brainly.com/question/9123296
#SPJ11
A professional athlete wants to tile his bedroom in solid gold. Each square tile will be 16
inches long and 1/4 inches thick. If the density of gold is 11. 17 ounces per cubic inch and
the price of gold is $1,303. 80 per ounce, how much will each tile cost? Round your
answer is the nearest dollar.
To calculate the cost of each tile, we need to first determine the volume of each tile. The length and width of the tile are given as 16 inches, and the thickness is given as 1/4 inches, which can be converted to 0.25 inches. Therefore, the volume of each tile is 16 x 16 x 0.25 = 64 cubic inches.
Next, we need to determine the weight of gold in each tile. Since the density of gold is 11.17 ounces per cubic inch, the weight of gold in each tile is 64 x 11.17 = 715.68 ounces.
Finally, we can calculate the cost of each tile by multiplying the weight of gold by the price of gold per ounce. The price of gold is given as $1,303.80 per ounce, so the cost of each tile is 715.68 x $1,303.80 = $933,526.78. Rounded to the nearest dollar, each tile will cost $933,527.
In summary, each square tile made of solid gold and measuring 16 inches long and 1/4 inches thick will cost approximately $933,527. This cost is based on the density of gold, which is 11.17 ounces per cubic inch, and the price of gold, which is $1,303.80 per ounce.
To know more about each tile, refer here
https://brainly.com/question/29906822#
#SPJ11
People were asked if they were considering changing what they eat.29% of the people asked said yes.of these, 23% said they were considering becoming vegetarian.what percentage of the people asked said they were considering becoming vegetarian?
Answer:
66.7%
Step-by-step explanation:
Let people asked bye x.
Then, people considering to change = 29% of x
People considering to become vegetarians = 23% of (29% of x)
= 23/100 * 29x/100
= 667x/10000
Percentage of people considering to become vegetarians = 667x/10
= 66.7%
Alejandro will deposit $1,750 in an account that earns 6. 5% simple interest every year. His sister Anallency will deposit $1,675 in an account that earns 7. 5% interest compounded annually. The deposits will be made on the same day, and no additional money will be deposited or withdrawn from the accounts. How much more money will Anallency have?
Anallency will have $134.46 more money than Alejandro after one year of their respective deposits.
How much more money will Anallency have than Alejandro after one year of their respective deposits?Alejandro and Anallency are depositing money in separate accounts with different interest rates. Alejandro is depositing $1,750 in an account that earns a simple interest rate of 6.5% per year, while Anallency is depositing $1,675 in an account that earns a compounded interest rate of 7.5% per year. After one year of their respective deposits, Anallency will have $134.46 more money than Alejandro.
The difference in interest earned between the two accounts is the reason for the difference in money earned by Alejandro and Anallency. Alejandro's account earns a simple interest rate, which means that he earns 6.5% of his initial deposit every year, regardless of how much interest he has already earned.
On the other hand, Anallency's account earns a compounded interest rate, which means that she earns interest on both her initial deposit and on any interest earned in previous years.
The formula for calculating simple interest is I = Prt, where I is the interest earned, P is the principal (or initial deposit), r is the interest rate, and t is the time in years. Using this formula, we can calculate that Alejandro will earn $113.75 in interest after one year.
The formula for calculating compounded interest is A = P(1 + r/n)^(nt), where A is the amount of money at the end of the time period, P is the principal, r is the interest rate, n is the number of times the interest is compounded per year, and t is the time in years.
Using this formula, we can calculate that Anallency will have $248.21 more in her account than Alejandro after one year.
Learn more about Interest rate
brainly.com/question/13324776
#SPJ11
one serving of almonds is 1/3 cup. booker bought 2 and 2/3 cups of almonds how many servings of almonds did booker buy
Booker bought 8 servings of almonds.
How many servings of almonds did Booker buy if he purchased 2 and 2/3 cups of almonds, and one serving of almonds is 1/3 cup?
One serving of almonds is equal to 1/3 cup.
To find how many servings of almonds Booker bought, we can divide the total amount of almonds he purchased by the amount in one serving:
2 and 2/3 cups of almonds = 8/3 cups of almonds
Number of servings = (total amount of almonds purchased) / (amount in one serving)
Number of servings = (8/3) / (1/3)
Number of servings = 8/3 x 3/1
Number of servings = 8
Learn more about servings
brainly.com/question/31639498
#SPJ11
Mrs. Hinojosa had 75 feet of ribbon. If each of the 18 students in her
class gets an equal length of ribbon, how long will each piece be?
Write your answer in 3 ways:
a. using only feet
b. using a whole number of feet and a whole number of inches
c. using only inches
Using division operation with unit conversions, the length of ribbon that each of the 18 students in Mrs. Hinojosa's class gets is as follows:
a) 4.2 feet.
b) 4 feet and 2 inches
c) 50 inches.
What is division operation?Division and multiplication operations are used in unit conversions.
Unit conversions involve converting measurements from hours to minutes or seconds, centimeters to meters and miles, etc.
The total quantity of ribbon Mrs. Hinojosa had = 75 feet
1 foot = 12 inches
75 feet = 900 inches (75 x 12)
The number of students in the class = 18
The length of ribbon received by each student = 4.167 feet (75 ÷ 18)
The length of ribbon received by each student ≈ 4 feet and 2 inches
The length of ribbon received by each student in inches only = 50 inches (900 ÷ 18)
Learn more about unit conversions, division, and multiplication at https://brainly.com/question/4158962.
#SPJ1
Be Precise The base of a triangle is 2 ft. The
height of the triangle is 15 in. What is the area
of the triangle in square inches?
Thus, the area of triangle for the given values of height and base is found as: 180 sq. in.
Explain about the conversion units:A number of steps are involved in the Unit of Conversion process, which involves multiplying or dividing by a numerical factor. There are numerous ways to measure things like weight, separation, and temperature.
Unit conversion is the process of changing the unit of measurement for a comparable quantity by multiplying or dividing by conversion factors.
Scientific notation is used to express the units, which are then translated into numerical values in accordance with the amounts.
Given data:
base of triangle b = 2 ft.Height h = 15 in.We know that,
1 foot = 12 in.
2 feet = 12*2 = 24 in.
Area of triangle = 1/2 * b * h
Area of triangle = 1/2 * 24 * 15
Area of triangle = 12* 15
Area of triangle = 180 sq. in
Thus, the area of triangle for the given values of height and base is found as: 180 sq. in.
know more about the conversion units:
https://brainly.com/question/13016491
#SPJ1
25. a state study on labor reported that one-third of full-time teachers in the state also worked part time at another job. for those teachers, the average number of hours worked per week at the part-time job was 13. after an increase in state teacher salaries, a random sample of 400 teachers who worked part time at another job was selected. the average number of hours worked per week at the part-time job for the teachers in the sample was 12. 5 with standard deviation 6. 5 hours. is there convincing statistical evidence at the level of 0. 05, that the average number of hours worked per week at part-time jobs decreased after the salary increase? (a) no. the p-value of the appropriate test is greater than 0. 5. (b) no. the p-value of the appropriate test is less than 0. 5. (c) yes. the p-value of the appropriate test is greater than 0. 5. (d) yes. the p-value of the appropriate test is less than 0. 5. (e) not enough information is given to determine whether there is convincing statistical evidence
From the solution to the question that we have here, the answer is A. There is no solid proof that the number of hours worked dropped after the income rise.
Let μ be the population mean number of hours worked per week by teachers who work part-time jobs, after the salary increase. Here are the alternate and null hypotheses:
H0: μ = 13
H1: μ < 13
n = 400
[tex]\bar{X}=12.5[/tex]
μ = 13
Formula for the t-test statistics is
[tex]t = \frac{\bar{X}-\mu}{s/\sqrt{n} }[/tex]
t = (12.5 - 13)/(6.5/√400)
t = (- 0.5)/(6.5/20)
t = - 10/6.5
= -1.5388
Degree of freedom is 400 -1 = 399
α = 0.05
The p-value is p(t < -1.5385) = 0.062
The p-value exceeds the level of significance. Therefore, we unable to reject the null hypothesis.
Hence, option a is correct.
Learn more on statistics here:
brainly.com/question/19243813
#SPJ4
Which number produces a rational number when multiplied by 1?
O A. TI
O в. -3
• C. Т
O D. -1. 41421356
Number that produces a rational number when multiplied by 1 is option B, -3.
To determine which number produces a rational number when multiplied by 1,
We need to examine each of the given.
Here, options: A. TI , B. -3, C. T, D. -1.41421356
We know,
A rational number can be expressed as a fraction (a/b) where both a and b are integers and b is not equal to 0. When multiplying by 1, the result remains the same. Option B: -3 multiplied by 1 = -3 which is a rational number because it can be expressed as a fraction (-3/1).
Option D: here given number -1.41421356 multiplied by 1 = -1.41421356 and we cannot be easily expressed as a fraction.
Therefore,it's not a rational number.
Thus, the number that produces a rational number when multiplied by 1 is option B, -3.
Learn more about rational number here,
https://brainly.com/question/12088221
#SPJ4
A three digit number is such that twice the hundreds digit is more than the tens digit by 2. The unit digit is thrice the hundred digit. When the digits are reversed the number is increased by 594. Find the number.(5 marks)
Answer:
Step-by-step explanation:
Let the three-digit number be represented as $abc$, where $a$ is the hundreds digit, $b$ is the tens digit, and $c$ is the units digit.
From the problem, we have two equations:
Equation 1: $2a=b+2$
Equation 2: $c=3a$
We can use these equations to solve for $a$, $b$, and $c$.
Starting with Equation 1, we can isolate $b$ to get $b=2a-2$.
Next, we can substitute Equation 2 into Equation 1 to get $2a=3a-6+2$, which simplifies to $a=8$.
Using this value of $a$, we can now find $b$ and $c$. From Equation 2, we have $c=3a=24$. And from Equation 1, we have $b=2a-2=14$.
Thus, the original three-digit number is $abc=824$.
When we reverse the digits to get $cba=428$, we increase the number by 594, so we have $cba=abc+594=824+594=1418$.
Therefore, the answer is $\boxed{824}$.
FILL IN THE BLANK. Find the maximum and minimum values of f(x, y) = xy on the ellipse 3x² + y² = 9. maximum value =_____ minimum value =_____
Maximum and Minimum values of f(x, y) = xy on the ellipse 3x² + y² = 9.
The maximum value is 3
The minimum value is -3
To find the maximum and minimum values of f(x, y) = xy on the ellipse 3x² + y² = 9, follow these steps:
how to find maximum and minimum value:1. Use the constraint equation (ellipse equation) to solve for one of the variables, either x or y.
Here, let's solve for y:
y² = 9 - 3x²
y = ±√(9 - 3x²)
2. Substitute y in the function f(x, y) with the expressions found in step 1:
f(x, y) = x(±√(9 - 3x²))
3. Differentiate f(x, y) with respect to x to find critical points (maximum or minimum):
f'(x, y) = ±(√(9 - 3x²) - (3x² / √(9 - 3x²)))
4. Set f'(x, y) = 0 and solve for x:
√(9 - 3x²) - (3x² / √(9 - 3x²)) = 0
5. Find the corresponding y values for the x values found in step 4 by substituting x back into the expressions found in step 1.
6. Evaluate f(x, y) at each critical point (x, y) found in steps 4 and 5 to determine the maximum and minimum values.
The maximum value of f(x, y) = xy on the ellipse 3x² + y² = 9 is 3, and the minimum value is -3.
To know more about Maximum values:
https://brainly.com/question/14316282
#SPJ11
PLEASE HELP Solve for f(x)!!
Answer:
8.81
Step-by-step explanation:
Substitute x for 7 and then solve normally
{2(7)^2+7-8}/(7)+4
{(2x49)+7-8}/11
98+7-8/11
97/11
8.81
The bike sarah wants to buy is now 40% off. the original price is $150. decide if you are missing the percent, part or whole. then use the appropriate formula to find the discount amount
The discount amount of the bike Sarah wants to buy is $60. The calculation was done by using the formula: Discount = Original price x Percent off.
To find the discount amount of the bike, we need to use the formula
Discount = Original Price x Discount Rate
where Discount Rate = Percent Off / 100
We know that the original price of the bike is $150 and it is now 40% off. So, the discount rate is
Discount Rate = 40 / 100 = 0.4
Substituting these values in the formula, we get:
Discount = $150 x 0.4 = $60
Therefore, the discount amount of the bike is $60.
To know more about discount amount:
https://brainly.com/question/29181625
#SPJ4
Find the length of the midsegment of the trapezoid
2n-2
N+12
4n+6
Answer: To find the length of the midsegment of a trapezoid, you need to add the lengths of the two bases together and divide by 2.
The length of the top base is 2n-2 and the length of the bottom base is 4n+6.
Adding the two bases together gives:
(2n-2) + (4n+6) = 6n + 4
Dividing by 2 gives:
(6n + 4) / 2 = 3n + 2
Therefore, the length of the midsegment of the trapezoid is 3n + 2.
If my refrigerator uses 300 watts when the motor is running, and the motor runs for 30 minutes of every hour, then how much energy does it use per day? How many BTU of energy would that be equal to?
The amount of energy the refrigerator uses per day is 3.6 kilowatt-hour (kWh)
The refrigerator energy usage per day in BTU is 12283.704 BTU
What is the BTU?The BTU is an acronym for the British Thermal Unit, which is a measure of heat, which is specified in energy units.
The duration the refrigerator fan runs per hour = 30 minutes
The amount of energy the refrigerator uses every hour = 300 watts × 0.5 hour/hour = 150 watts
The amount of energy the refrigerator uses per hour = 150 watt-hour
The amount of energy the refrigerator uses per day = 24 × 150 watt-hour = 3600 watt-hour = 3.6 kilowatt-hour (kWh)1 kWh = 3412.14 BTU
Therefore;
3.6 kWh = 3.6 × 3412.14 BTU = 12283.704 BTUThe amount of energy in BTU the refrigerator uses per day = 12283.704 BTU
Learn more on the BTU here: https://brainly.com/question/14252954
#SPJ1
If the shapes are scaled copy select a reasonable scale factor that could be applied to shape to 2 create shape 1. i need help asap
A reasonable scale factor that could be applied to Shape 2 to create Shape 1 is 0.5.
What scale factor can be used to transform Shape 2 into Shape 1, if they are scaled copies?When we talk about scaling a shape, we mean changing the size of the shape while maintaining its overall proportions.
This can be done by multiplying all of the dimensions of the shape by the scale factor.
For example, if we wanted to make a shape twice as big, we would multiply all of its dimensions (length, width, and height) by 2. If we wanted to make it half as big, we would multiply all of its dimensions by 0.5.
Looking at the two shapes, we can see that Shape 1 is half the size of Shape 2 in all dimensions.
For example, the height of Shape 1 is half the height of Shape 2, the width of Shape 1 is half the width of Shape 2, and the length of Shape 1 is half the length of Shape 2.
Therefore, to transform Shape 2 into Shape 1, we need to multiply all of its dimensions by 0.5. This will result in a scaled copy of Shape 2 that is identical in shape to Shape 1.
Learn more about scale factor
brainly.com/question/30215119
#SPJ11
Lourdes could choose to pay an ATM fee to get cash or use a credit card to pay for groceries
that cost $48. The credit card charges interest if the balance is not paid at the end of the month.
Should she use the debit card or credit card? Justify your choice.
Keep in mind that if Lourdes is able to pay off her credit card balance at the end of the month, she won't be charged any interest, making the credit card the better option in that case.
To determine whether Lourdes should use a debit card or credit card, we need to consider the ATM fee and the potential interest charges from the credit card.
Step 1: Determine the ATM fee
Find out how much Lourdes would be charged for using the ATM to withdraw cash.
Step 2: Determine the interest rate on the credit card
Check the credit card's terms and conditions to find the annual percentage rate (APR). This will help us calculate the potential interest charges.
Step 3: Calculate the potential interest charges
Assuming Lourdes doesn't pay off the credit card balance by the end of the month, divide the APR by 12 to get the monthly interest rate. Multiply this rate by the $48 grocery cost to find the potential interest charges.
Step 4: Compare costs
Compare the ATM fee and potential interest charges to determine the cheaper option. If the ATM fee is less than the potential interest charges, Lourdes should use her debit card. If the potential interest charges are less than the ATM fee, she should use her credit card.
Keep in mind that if Lourdes is able to pay off her credit card balance at the end of the month, she won't be charged any interest, making the credit card the better option in that case.
Learn more about credit card,
https://brainly.com/question/26857829
#SPJ11
Reflect (-5,-3) over the x axis then translate the result 2 units downwhat are the final cordinates
We reflect the point (-5, -3) over the x-axis to get (-5, 3), and then translate this point down by 2 units to get the final coordinates of (-5, 1).
To reflect a point over the x-axis, we keep the x-coordinate the same, but negate the y-coordinate. Thus, reflecting the point (-5, -3) over the x-axis gives us the point (-5, 3).
Next, we need to translate the result 2 units down. To do this, we subtract 2 from the y-coordinate of the reflected point. So, subtracting 2 from the y-coordinate of (-5, 3) gives us the final coordinates:
(-5, 3) translated 2 units down is (-5, 1).
In summary, we can reflect a point over the x-axis by negating the y-coordinate, and translate a point down by subtracting a value from its y-coordinate. So, we reflect the point (-5, -3) over the x-axis to get (-5, 3), and then translate this point down by 2 units to get the final coordinates of (-5, 1).
To know more about reflection, refer to the link below:
https://brainly.com/question/16359605#
#SPJ11
Terrell arranges x roses at $3. 50 each with 10 carnations at $2. 25 each. He makes a bouquet of flowers that averages $3. 00 per flower. Choose an equation to model the situation
The equation models the situation described in the problem is 3.50x + 2.25(10) = 3 (x + 10) . The correct answer is C.
To model the situation described in the problem, we need to use an equation that represents the total cost of the flowers in terms of the number of roses (x) and the number of carnations (10). Let's assume that the cost of each flower is proportional to its price, and that the average cost per flower is the total cost divided by the total number of flowers (x + 10).
The cost of x roses at $3.50 each is 3.50x, and the cost of 10 carnations at $2.25 each is 2.25(10) = 22.50. Therefore, the total cost of the bouquet is:
Total cost = 3.50x + 22.50
The average cost per flower is given by:
Average cost = Total cost / (x + 10)
We are told that the average cost per flower is $3.00, so we can set up an equation:
3.00 = (3.50x + 22.50) / (x + 10) or 3.50x + 2.25(10) = 3 (x + 10)
This equation models the situation described in the problem. We can solve for x to find the number of roses needed to make a bouquet that meets the given conditions. The correct answer is C.
Your question is incomplete but most probably your full question attached below
To learn more about equation visit:
brainly.com/question/21835898
#SPJ11
Selika give her garden a makeover. She spends money on plant,materials, and labour in the ratio of 1:5:12. She spends £848. 75. How much money does she spend on labour costs
Selika spends £565.85 on labour costs.
Given, Selika spends money on plants, materials, and labor in the ratio of 1:5:12 and spends a total of £848.75. We have to find how much money she spends on labor costs.
Let the amount of money Selika spends on plants be x. Then, the amount of money she spends on materials is 5x, and the amount of money she spends on labor is 12x.
The total amount of money she spends is £848. 75
x + 5x + 12x = 848.75
18x = 848.75
x = 848.75/18
x = 47.15
She spend on labour 12x = 12 × 47.15
= 565.85
Therefore, Selika spends £565.85 on labour costs.
Learn more about ratio here
brainly.com/question/13419413
#SPJ4
Which set includes ONLY rational numbers that are also integers?
The set that includes ONLY rational numbers that are also integers is:
{-3, -2, -1, 0, 1, 2, 3, ...}
Which set includes ONLY rational numbers also integers?The set of rational numbers that are also integers is the set of numbers that can be expressed as a ratio of two integers where the denominator is 1. This means that the set includes numbers that are whole numbers, as well as their negatives.Learn more about integers
brainly.com/question/27908445
#SPJ11
Here is an inequality: -2x > 10.
1. List some values for x that would make this inequality true.
2. How are the solutions to the inequality -2x [tex]\geq[/tex] 10 different fomt the solutions to -2x > 10? Explain your reasoning.
Therefore , the solution of the given problem of inequality comes out to be the solutions x = -6 or x = -10 would be acceptable.
What exactly is an inequality?Algebra, which lacking a symbol for this difference, can represent it using a pair or group of numbers. Equity usually comes after equilibrium. Inequality is bred by the persistent gap of standards. Equality and disparity are not the same thing. As was my least preferred symbol, notwithstanding knowing that the pieces are often not connected or close to one another. (). No matter how small the variations, they all affect value.
Here,
Finding values of x that cause the left side of the inequality to be bigger than the right side is necessary to make the inequality -2x > 10 true. Divide both sides by -2 and invert the inequality sign to achieve this:
=> -2x > 10
=> x < -5
The inequality is therefore true for any value of x that is less than -5. For instance, the solutions x = -6 or x = -10 would be acceptable.
To know more about inequality visit:
https://brainly.com/question/29914203
#SPJ1
Please help!!!!!!
Carillo Industries collected $108,000 from customers in 2020. Of the amount collected, $25,000 was for services performed in 2019. In addition, Carillo performed services worth $36,000 in 2020, which will not be collected until 2021. Carillo Industries also paid $72,000 for expenses in 2020. Of the amount paid, $30,000 was for expenses incurred on account in 2019. In addition, Carillo incurred $42,000 of expenses in 2020, which will not be paid until 2021.
Whatâs the cash basis income and the accrual basis net income?
The cash basis income for Carillo Industries is $41,000 and the accrual basis net income is $35,000.
How are the income bases?
The cash basis income for Carillo Industries, based on the cash received and paid during the year, is $41,000. The accrual basis net income, based on revenue and expenses earned/incurred during the year, regardless of whether cash was exchanged, is $35,000.
In 2020, Carillo Industries collected $108,000 from customers, of which $25,000 was for services performed in 2019 and $36,000 will not be collected until 2021.
The company paid $72,000 for expenses in 2020, of which $30,000 was for expenses incurred in 2019, and $42,000 of expenses in 2020 will not be paid until 2021. By calculating the cash and accrual basis, Carillo Industries can understand its financial performance from different perspectives.
Learn more about Carillo Industries
brainly.com/question/16891177
#SPJ11
Consider the vector field.
F(x, y, z) =
5ex sin(y), 9ey sin(z), 2ez
sin(x)
(a) Find the curl of the vector field.
curl F =
(b) Find the divergence of the vector field.
div F =
(a) To find the curl of the vector field F(x, y, z), we first find its component functions:
F(x, y, z) = (5ex sin(y), 9ey sin(z), 2ez sin(x))
Then, we use the formula for the curl of a vector field:
curl F = (∂Fz/∂y - ∂Fy/∂z, ∂Fx/∂z - ∂Fz/∂x, ∂Fy/∂x - ∂Fx/∂y)
Plugging in the component functions of F(x, y, z), we get:
curl F = (2ez cos(x), -5ex cos(y), 9ey cos(z))
(b) To find the divergence of the vector field F(x, y, z), we use the formula for the divergence of a vector field:
div F = ∂Fx/∂x + ∂Fy/∂y + ∂Fz/∂z
Plugging in the component functions of F(x, y, z), we get:
div F = 5e^x sin(y) + 9e^y sin(z) + 2e^z sin(x)
(a) To find the curl of the vector field F(x, y, z) = (5e^x sin(y), 9e^y sin(z), 2e^z sin(x)), we need to compute the cross product of the del operator (∇) and F:
curl F = ∇ x F
curl F = ( (∂/∂y)(2e^z sin(x)) - (∂/∂z)(9e^y sin(z)), (∂/∂z)(5e^x sin(y)) - (∂/∂x)(2e^z sin(x)), (∂/∂x)(9e^y sin(z)) - (∂/∂y)(5e^x sin(y)) )
After computing the partial derivatives, we get:
curl F = ( 0, 5e^x cos(y) - 2e^z cos(x), 9e^y cos(z) - 5e^x cos(y) )
(b) To find the divergence of the vector field F(x, y, z), we need to compute the dot product of the del operator (∇) and F:
div F = ∇ ⋅ F
div F = (∂/∂x)(5e^x sin(y)) + (∂/∂y)(9e^y sin(z)) + (∂/∂z)(2e^z sin(x))
After computing the partial derivatives, we get:
div F = 5e^x sin(y) + 9e^y sin(z) + 2e^z sin(x)
learn more about vector field here: brainly.com/question/16180391
#SPJ11
Rotation of 180°, followed by a dilation with scale factor 5, followed by a reflection over the line y = x.
a. A' (15, -10) b.
A' (-15, 10)
C. A' (-10, 15)
d. A' (10, -15)
Answer:
A
Step-by-step explanation:
Let's call the length of each of the other two sides x. Since the triangle is isosceles, it has two sides of equal length. Therefore, the perimeter of the triangle can be expressed as 6 + x + x Simplifying this equation, we get 2x + 6 We know that the perimeter is 22 cm so we can set up an equation and solve for x. 22 = 2x + 6 Subtracting 6 from both sides, we get 16 = 2x Dividing both sides by 2, we get x=8
Draw the image of ABC under the translation by 1 unit to the right and 4 units up
The image of ABC under the translation by 1 unit to the right and 4 units up is the triangle with vertices A', B', and C'.
What is Triangle?A triangle is a closed two-dimensional geometric shape with three straight sides and three angles. It is one of the basic shapes in geometry and has been studied since ancient times. To translate a figure by a given vector, you need to move each point of the figure by the same amount and in the same direction as the vector. In this case, we want to translate ABC by a vector of (1, 4), which means we need to move each point of ABC one unit to the right and four units up.
Let's say the coordinates of the points of ABC are:
A = [tex]\rm (x_1, y_1)[/tex]
B = [tex]\rm (x_2, y_2)[/tex]
C = [tex]\rm (x_3, y_3)[/tex]
To translate ABC by the vector (1, 4), we add 1 to each x-coordinate and 4 to each y-coordinate:
A' = [tex]\rm (x_1 + 1, y_1 + 4)[/tex] (x₁ + 1, y₁ + 4)
B' = [tex]\rm (x_2 + 1, y_2 + 4)[/tex] (x₂ + 1, y₂ + 4)
[tex]\rm C' = (x_3 + 1, y_3 + 4)[/tex] (x₃ + 1, y₃ + 4)
Therefore, The image of ABC under the translation by 1 unit to the right and 4 units up is the triangle with vertices A', B', and C'.
To learn more about Triangle from given link.
https://brainly.com/question/30599944
#SPJ1